Presentation is loading. Please wait.

Presentation is loading. Please wait.

1.Which one of the following statements about the treatment of depression is correct:  A. Selective serotonin reuptake inhibitors (SSRIs) should be used.

Similar presentations


Presentation on theme: "1.Which one of the following statements about the treatment of depression is correct:  A. Selective serotonin reuptake inhibitors (SSRIs) should be used."— Presentation transcript:

1 1.Which one of the following statements about the treatment of depression is correct:
 A. Selective serotonin reuptake inhibitors (SSRIs) should be used with caution in the first few weeks of treatment because they have been associated with an increased rate of completed suicides B.SSRIs are associated with more adverse effects than tricyclic antidepressants.   C. Venlafaxine is more effective than tricyclic antidepressants.   D. Cognitive therapy has been shown to effectively improve depressive symptoms.

2 The answer is: D Treatment: Always start with an SSRI SSRI increase the risk of Suicide (Ideation, attempted, complete) First Neurovegetative Sx 1-3 weeks next Emotional Sx Serotonin Syndrome, switching from an SSRi to MAOI…Hyperthermia, Myoclonus, rigor and Hypertonicity Other Treatments: ECT (Study The indications), Psychotheray: Cognetive Therapy

3 2. In treating manic episodes, which of the following is most useful in the first three days of treatment? A. Haloperidol 5 mg bd B. Chlorpromazine 10 mg po bd C. Lithium 300 mg od D. Carbamazepine 200 mg bd E. A quit, dimly-lit room to decrease stimulation

4 The answer is: A Mood Stabilizers: Antipsychotics:
Lithium: Therapeutic Response within 7-14 days up to 3 weeks, mg/day - ) mmol/L ( mmol/L Acute) – Monitoring: Biweekly or Monthly, q2months – Thyroid and Creatinine 6 months, UA annual Carbamazepine (Tegretol): mg/day Rapid cycling, Severe sleep problems, Can't take Depakote lamotrigine/Lamictal: Depression is the dominant symptom, Dysphoric mood Valproic Acid (Depakene-Epival) valproate/Depakote: Need something strong and fast, Male gender, and not afraid of weight gain risk Rapid cycling, Significant manic symptoms Antipsychotics: Haloperidol: PO b/tid, Chlorpromazine (largactil): PO b/tid FDA:To control the manifestations of the manic type of manic-depressive illness, low potency Olanzapine/Zyprexa: Emergency-level symptoms, Need help really fast,Can use on "as-needed" basis

5 3. A 22 yr old girl with history of vomiting, loss of weight and amenorrhea. History compatible with anorexia nervosa. At present what are the investigations you will do, except: A. Serum electrolyte B. Liver function test C. Gonadotrophin levels. D. ECG E. bone densitometry

6 The answer is: E A chemistry panel should be assessed for hypokalemic, hypocalcemic metabolic alkalosis caused by vomiting. Ionized calcium levels should detect hypocalcemia Liver function tests may be slightly elevated, but albumin and protein levels are usually normal Thyroid function tests, prolactin, and serum follicle-stimulating hormone levels can differentiate anorexia nervosa from alternative causes of primary amenorrhea. ECG is helpful in evaluating the severity of malnutrition and risk for dysrhythmias in patients with metabolic abnormalities Imaging is rarely necessary in the emergency department

7 4.Pregnant woman on SSRI-what will you do?
A. Continue B. Switch to something else C. Discontinue D. Tell her the risk is low

8 The answer is: A Advisory March 10, 2006 For immediate release OTTAWA - Health Canada is advising women who are taking antidepressants known as Selective Serotonin Re-uptake Inhibitors (SSRI) and who are pregnant or intend to become pregnant to discuss the situation with their doctor due to potential risks to the baby. Health Canada stresses that patients should not stop taking SSRI medication without first consulting their doctors, as they could experience serious side effects. Generally, SSRI treatment should only be continued if the benefits to the individual patient are thought to outweigh the risks to the unborn child, while also considering the benefits and risks of switching to another treatment option or stopping treatment altogether. These precautions and the possibility of adverse health effects in newborns are mentioned in the current Canadian prescribing information and consumer information for SSRIs SSRI are Class C = risk to fetus cannot be ruled out Electroconvulsive therapy has been used to treat depression in pregnancy for over 50 years. This technique has been reported to be relatively safe in pregnant women with severe, refractory depression

9 5.What is the minimum age that you can start an insight-psychoanalysis with a child?
A.6Y B.10Y C.14Y D.17Y

10 The answer is: A Anna Freud Centre, 21 Maresfield Gardens, London NW3 5SD. After a discussion of some of the theoretical issues raised in the past about the concept of insight, the authors go on to track changes in the internal representations of two young patients during the course of a session with each of them. Their argument is that in the attention to the detail of these sessions there is evidence of a gradual development of knowledge about internal relationships in the patient, which is reflected in movements in the narrative of the play. The need for congruence between heightened affect in the patient and the analyst conveying understanding is stressed and compared with early mother-infant relationships. They argue that the 5-year-old child can use his developing capacity to mentalise most safely in the context of pretend play and that the analyst's communications should remain as close as possible to that context, typically in displacement. They conclude that their sessions give evidence that children of this age are capable of a form of insight that could be said to be the equivalent of that of adults in analysis, but that their mode of expression and of communication of their self-knowledge through their play is very different.

11 6.Concerning childhood psychiatric disorder each of the following is correct except:
A. Children with mild MR are identifiable at 2-3 years of age B. Children with developmental reading disorder have a normal intelligence C. Presence of autism in siblings of autistic children is 50 times higher than in general population D. Diagnosis of encopresis can be made when a child is 4 year old E. Children with gender identity disorder rarely develop transsexualism in adult life

12 The answer is: A For clinically referred children, onset of cross-gender interests and activities is usually between ages 2 and 4 years, and some parents report that their child has always had cross-gender interests. Only a very small number of children with Gender Identity Disorder will continue to have symptoms that meet criteria for Gender Identity Disorder in adolescence or adulthood most children with mental retardation do not develop symptoms that are noticeable until the preschool period. Symptoms become apparent at a younger age in those more severely affected. Usually, the first problem parents notice is a delay in language development.

13 7.An 18 yrs girl with good health previously, seeks helps at an emergency room for lightheadedness, headaches and nausea. She appears anxious and is tremulous. sweating and breathing heavily. While waiting for doctor she began to complain of tingling around her mouth and her fingers. which one 1st step of management? A. Ask her breath into a paper bag B. Immediate IV glucose solution. C. Order a urine drug screen D. Give oxygen with mask E. 2mg of SL lorazepam (ativan)

14 The answer is: A Panic Attack: Abruptly and peak within 10 min.
Management: Box-Breathing, Relaxation Techniques Supportive Psychotherapy, CBT BZD (Alprazolam), SSRI, SNRI except Wellbutrin

15 8.Which of the following is correct to anorexia nervosa:
A. Significant depression usually occurs B. Appetite is not initially lost C. Pursuit of thinness becomes the only priority D. Loosing weight becomes associated with an escalation of anger E. Purging often develops later in the course of the illness

16 The answer is: C A refusal to maintain body weight at or above a minimally normal weight for age and height (usually less than 85% of ideal body weight) Intense fear of gaining weight or becoming fat Disturbance in the way one's body weight or shape is experienced, with denial of current low body weight Amenorrhea in postmenarcheal females of at least 3 menstrual cycles Restricting Type Binge-Eating-Purging Type

17 9.All of the following conditions are included in the DDE of anxiety disorder except:
A. Schizophrenia B. Hypothyroidism C. Panic Disorder D. DM

18 The answer is: A DDE of anxiety Disorders
Endocrine: Hypo- or Hypothyroidism, Pheocromocytoma, hypoglacemia, hyperadrenalism, Hyperpara CVS: MVP, CHF, PE, Arrhythmia Respiratory: Astham, Pneumonia, Hyperventilation Metabolic: Vit B12 Def., Porphyria Neurologic: Neoplasm, Vestibular Dysfunction, Encephalitis Substance Abuse, Withdrawal

19 9.65 yr old woman living alone in a dilapidated house, although her family members have tried in vain to move her to a better dwelling. She wears odd clothes, rummages in the garbage cans of neighbors to look for redeemable cans and bottles. she is very suspicious of her neighbors and is convinced that they were trying to end her. she believes in the "power of crystals" and had them scattered all over her house. diagnosis: A. Schizophrenia, paranoid B. Schizotypal personality disorder C. Avoidant personality disorder D. Schizoid personality disorder E. Borderline disorder

20 The answer is: B Schizotypal PD: Odd Thinking and Speech Patterns, Odd and eccentric Behavior. Paranoid Ideation and magical thinking or odd beliefs Avoidant PD: Avoids occupational activities that involves significant criticism or rejection Schizoid PD: Neither Desires nor enjoys close relationships Borderline PD: Unstable Affect, mood Swings, Marked Impulsivity, unstable relationship Schizophrenia, paranoid: Delusions of persecution and Grandeur. In late 20s or 30s, best prognosis

21 10.A patient presents to your office with a history of ongoing excessive alcohol use and associated depression.  Best recommendations could include all of the following EXCEPT: A. Disulfiram 250mg po qdaily B. Brief intervention for alcohol C. Naltrexone 50mg po qhs D. Cognitive behavioral therapy E. Increased follow-up

22 The answer is: A The first step in treatment is brief intervention. The physician states unequivocally that the patient has a problem with alcohol and emphasizes that this determination stems from the consequences of alcohol in that patient's life, not from the quantity of alcohol consumed. Emphasizing the effects on family, friends, and occupation, as well as any physical manifestations, is important. Pointing out that loss of control and compulsive use indicate alcohol dependence also is important. Naltrexone blocks opiate receptors and works by decreasing the craving for alcohol, resulting in fewer relapses. Most, but not all, studies found that naltrexone decreases relapses but the effect is modest (12-20%). Combining naltrexone therapy with cognitive behavioral therapy enhanced benefit. One study showed benefit with an intensive primary care intervention. Studies suggest that virtually all placebo patients who sampled alcohol relapsed, while only half the naltrexone patients who sampled alcohol relapsed. A number of medications have been tried in the treatment of alcoholism. Disulfiram (Antabuse) has been used as an adjunct to counseling and AA with motivated patients to reduce the risk of relapse. Patients are reminded of the risks of adverse effects when tempted to drink. Disulfiram causes nausea, vomiting, and dysphoria with coincident alcohol use. In a large trial, disulfiram did not increase abstinence. If a patient asks for disulfiram and thinks it will help, it might be worth considering.

23 Disulfiram produces a sensitivity to alcohol which results in a highly unpleasant reaction when the patient under treatment ingests even small amounts of alcohol. Disulfiram plus even small amounts of alcohol produces flushing, throbbing in head and neck, throbbing headache, respiratory difficulty, nausea, copious vomiting, sweating, thirst, chest pain, palpitation, dyspnea, hyperventilation, tachycardia, hypotension, syncope, marked uneasiness, weakness, vertigo, blurred vision, and confusion. In severe reactions, there may be respiratory depression, cardiovascular collapse, arrhythmias, myocardial infarction, acute congestive heart failure, unconsciousness, convulsions, and death. As an aid in the management of selected chronic alcoholic patients who want to remain in a state of enforced sobriety so that supportive and psychotherapeutic treatment may be applied to best advantage. Used alone, without proper motivation and without supportive therapy, disulfiram is not a cure for alcoholism, and it is unlikely that it will have more than a brief effect on the drinking pattern of the chronic alcoholic. Contraindications Patients who are receiving or have recently received metronidazole, paraldehyde, alcohol, or alcohol-containing preparations such as cough syrups, elixirs, should not be given disulfiram. Severe myocardial disease or coronary occlusion; diabetes mellitus; hepatic cirrhosis or insufficiency; hypothyroidism; epilepsy; cerebral damage; chronic and acute nephritis; psychoses Warnings Disulfiram should never be administered to a patient without his full knowledge or when he is in a state of alcohol intoxication. The attending physician should instruct those tending the patient accordingly. Patients must be fully informed about the disulfiram-alcohol reaction. They must be strongly cautioned against surreptitious drinking while taking the drug and must be fully aware of possible consequences. They should be warned to avoid alcohol in disguised form, i.e., in sauces, vinegars, cough and cold mixtures, and even aftershave lotions or liniments. They should also be warned that reactions may occur with alcohol up to 14 days after ingestion of disulfiram.

24 11. Loosening of associations is most often found in;
A. depression. B. schizophrenia. C. bipolar disorder. D. drug-induced delirium. E. transient global amnesia

25 The answer is: B loosening of association Transient global amnesia
A manifestation of a severe thought disorder characterized by the lack of an obvious connection between one thought or phrase and the next, or with the response to a question. Transient global amnesia A memory disorder seen in middle aged and elderly persons characterized by an episode of amnesia and bewilderment that persists for several hours; during the episode the patient has a memory defect for present and recent past events, but is fully alert, oriented, capable of high-level intellectual activity, and has a normal neurological examination. Typically, these amnesic episodes occur spontaneously, and most patients experience only one; of uncertain etiology—probably ischemic, but not due to atherosclerosis.

26 12.In the emergency room you are asked to assess a man with alcoholic cirrhosis and recent alcohol cessation.  He is agitated, confused and hallucinating.  You make a presumptive diagnosis of delirium tremens.  What is the best treatment for his condition? A.    diazepam (Valium®)      B.    nitrazepam (Nitrazedon®)      C.    chlordiazepoxide (Librium®)      D.    lorazepam (Ativan®)      E.    clonazepam (Rivotril®)

27 The answer is: D Because of rapid onset, prolonged duration of effects, and high therapeutic index, Diazepam is drug of choice. Volumes of literature exist regarding usage of diazepam for ethanol withdrawal. Onset of action is within a couple of min after IV administration. Has active metabolite (desmethyl-diazepam) that has longer duration of action than diazepam. Severe Liver Disease, severe asthama, respiratory failure present  Lorazepam Although diazepam and chlordiazepoxide Both diazepam and chlordiazepoxide undergo hepatic metabolism and produce pharmacologically active metabolites. Both parent drug and metabolites result in prolonged half-lives and may accumulate in the elderly or patients with liver disease. Lorazepam and midazolam have the advantages of short action and ease of titration, and they produce no active metabolites. Lorazepam offers many advantages over diazepam and chlordiazepoxide, especially in elderly and hepatically dysfunctional patients

28 13.Patient comes to your office brought by her spouse after surviving a mugging where she was punched in the face. Three weeks later, her contusions are better but she is afraid to leave the house out of fear of another attack. Her sleep is disrupted by recurrent intrusive dreams of further attack. She is on edge, startles easily, and is easily moved to tears. The most likely diagnosis is? A. Panic disorder with agoraphobia B. Specific phobia C. Post traumatic stress disorder D. Transient stress induced anxiety E. Generalized anxiety disorder

29 The answer is: D A) Panic Disorder: are recurrent unexpected panic attacks, with four of the 13 symptoms that could be present, reach a peak within 10 minutes in the presence of intense fear. And, at least one of the attacks has been followed by one month of persistent concern about having additional attacks and/ or worry about the implications of the attack, such as fear of having a heart attack or going crazy. B) Specific Phobia: Marked and persistent fear that is excessive or unreasonable, that is caused by the presence or anticipation of a specific object or situation. Usually, they don’t have anxiety the rest of the time. And, in individuals under 18, the duration must be at least six months. C) Post-Traumatic Stress Disorder: Occurs after an individual has been exposed to a traumatic event that is associated with intense fear or horror. The patient rR-experiences the event through intrusive recollection or nightmares, flashbacks, or intense distress when exposed to reminders of the event. And, the symptoms have been present for at least one month. D) Transient stress induced anxiety or Acute Stress Disorder: Symptoms occur after an individual has been exposed to a traumatic event that is outside the realm of normal human experience. Re-experiences the event as in PTSD. But the symptoms occur within one month of a stressor and last between 2 days and four weeks. E) Generalized Anxiety Disorder: The excessive anxiety or worry is present most days during at least a six-month period.

30 14.The best treatment for the patient in the above question is?
A. Fluoxetine 20mg po qdaily B. Link the assault to the patient’s anxiety symptoms and support them in talking about the assault C. Imovane 7.5mg po qhs D. Clonazepam 0.5mg po tid E. Dynamic psychotherapy – relate the assault and current symptoms to issues from their past 

31 The answer is: B Link the assault to the patient’s anxiety symptoms and support them in talking about the assault

32 15. Tardive Dyskinesia can be suppressed initially by;
A.  anticholinergics B.  stopping the antipsychotic C.  MAOI D.  increasing the antipsychotic E. TCA

33 The answer is: B 1. All neuroleptics, with the exception of Clozapine, produce tardive dyskinesia. The risk of tardive dyskinesia with atypical antipsychotics is substantially decreased compared to typical agents 2. Antiparkinsonian drugs are of no benefit for tardive dyskinesias and may exacerbate symptoms. 3. When tardive dyskinesia symptoms are observed, the offending drug should be discontinued. Patients who require continued neuroleptic therapy should be switched to an atypical agent or Clozapine (if severe). 4. The risk of tardive dyskinesia increases with the duration of neuroleptic exposure, and there is an incidence of 3% per year with typical agents. 5. Most patients have relatively mild cases, but tardive dyskinesia can be debilitating in severe cases. Tardive dyskinesias not always improve with discontinuation or lowering of the does of neuroleptic

34 YO woman is admitted to a medical unit because of a 20lb wt loss over the Previous two months. She also reports anorexia, fatigue and sever constipation. Physical examination, blood work and sigmoidoscopy are all normal. Which of the following would be the most appropriate management? A.   Continuing physical investigations to rule out organic causes B.   Amitriptyline C.   Lorazepam D.   High fiber diet E.   Desipramine

35 The answer is: E The symptomatic presentations of depression in late life are by and large similar to the presentations throughout adulthood. The differences in symptom presentation that often are attributed to increased age, such as pseudodementia and depression masked as physical illness, are usually secondary to comorbidity (i.e., depression associated with dementia or physical illness and therefore not related to age). However, depressed older adults may differ from the depressed in middle age in that they more often experience weight loss and less often report feelings of worthlessness and guilt. Although elderly people suffering from major depression perform less well on objective tests of cognitive functioning, they are no more likely to complain of memory problems and concentration than middle-aged depressives. Ref: Kelley's Textbook of Internal Medicine. Part 11. Geriatrics, Chapter Approach to the Elderly with Depression. In the Elderly the first choice of pharmacotherapy are tricyclic antidepressants (Nortriptyline or Desipramine). The SSRI are second line agents because of their long half-life and side effects (agitation, insomnia, weight loss Desipramine Agent of Choice in patients over 75 Amitriptyline CVS side effects in Older patients

36 17.You have a pt. with stimulant overdose, in his follow up which is the most helpful in preventing recurrence .... A. Naloxone B. methadone C. supporting him by psychotherapy D. supportive family

37 The answer is: C Cognitive-behavioral therapy is focused on the delivery of information and development of skills that, in theory, enable a patient to discontinue drug use and avoid relapse. The techniques used within the designation of the cognitive-behavioral therapy include psychoeducation, identification of high-risk situations and warning signs for relapse, development of coping skills, development of new lifestyle behaviors, increased self-efficacy, and dealing with relapse. Cognitive-behavioral therapy has been applied successfully across many different substances of abuse. Cognitive-behavioral therapy seems to be particularly effective among patients at higher levels of addiction severity. Ref: Sadock, Benjamin J., Sadock, Virginia A. Kaplan & Sadock's Comprehensive Textbook of Psychiatry, 8th Edition, Substance-Related Disorders Naloxene to Extinguish drug-seeking behavior

38 18.27-year-old woman seeks evaluation for her "depression" in an out-patient clinic.  She reports episodic feelings of sadness since adolescence.  Occasionally she feels good, but these periods seldom last more than 2 weeks.  She is able to work but thinks she is not doing as well as she should.  In describing her problems she seems to focus more on repeated disappointments in her life and her low opinion of herself than on discrete depressive symptoms.  In your differential diagnosis at this point, Which one of the following is the most likely diagnosis?   A.    major depression with melancholia  B.    adjustment disorder with depressed mood  C.    cyclothymia  D.    childhood depression  E.    dysthymia

39 The answer is: E In dysthymic disorder, depressive symptoms typically begin insidiously in childhood or adolescence and pursue an intermittent or low-grade course over many years or decades; major depressive episodes may complicate it (double depression). In pure dysthymia, depressive manifestations occur at a subthreshold level and overlap considerably with those of a depressive temperament: habitually gloomy, pessimistic, humorless, or incapable of fun; passive and lethargic; introverted; skeptical, hypercritical, or complaining; self-critical, self-reproaching, and self-derogatory; and preoccupied with inadequacy, failure, and negative events.

40 year-old, previously successful woman was climbing stairs in her new home about a month ago, when the whole house fell apart. She ended up in a hospital with a fractured left femur. The psychiatry team was consulted because the patient complained of nightmares and flashbacks and was afraid to go to sleep as a result. During the interview, she is tearful, and afraid that her fear of falling is preventing her from participating enough in her rehabilitation, and that the team will discharge her from hospital. Which of the following is the most appropriate treatment for this patient? A. Insight-oriented psychotherapy B. No therapy because the patient needs to take responsibility for her treatment C. Put a sitter to stay in the patient's room 24 hours a day to calm her anxiety D. Start an antidepressant E. Start benzodiazepines

41 The answer is: D The diagnosis in this patient is Post-Traumatic Stress Disorder Post- Traumatic Stress Disorder Diagnostic Criteria: The person has experienced or witnessed or was confronted with an unusually traumatic event that has both of these elements: The event involved actual or threatened death or serious physical injury to the person or to others, and the person felt intense fear, horror or helplessness. The person repeatedly relives the event in at least 1 of these ways: - Intrusive, distressing recollections - thoughts, images. - Repeated, distressing dreams. - Through flashbacks, hallucinations or illusions, acts or feels as if the event were recurring. - Marked mental distress in reaction to internal or external cues that symbolize or resemble the event. - Physiological reactivity - such as rapid heart beat, elevated blood pressure in response to these cues. The above symptoms have lasted longer than one month. The treatment is CBT (that is not mentioned in the options provided) and Antidepressants patients may experience guilt because they behaved aggressively and destructively during armed combat or because they survived a traumatic experience in which family members or close associates perished--so-called survivor guilt. In such cases, psychodynamic or insight-oriented psychotherapy aimed at helping patients understand and modify their self-critical and punitive psychologic attitudes may be helpful

42 20. an 84 year old man presents to the ER
20. an 84 year old man presents to the ER .his son tells the physician that his father has been very depressed in the past few months ,has been eating poorly, sleeping restlessly and complaining of decreased energy, Particularly in morning .3 days ago the man's family physician prescribed amitrptyline 75 mg for the first night and 100 mg on the second. This evening ,the son says, his father has been increasingly restless ,shouting incoherently and screaming ''get these things off me'‘ the most likely diagnosis that accounts for his deterioration is A. worsening depression B. paranoid disorder C. schizophrenia of old age D. delerium E. dementia

43 The answer is: D Older adults are especially liable to certain side effects of Elavil, including rapid heartbeat, constipation, dry mouth, blurred vision, sedation, and confusion, and are in greater danger of sustaining a fall.

44 21. which one of the following medications given IV would confirm the present diagnosis in this man?
A .atropine B. amitriptyline C. physostigmine D.  diazepam

45 The answer is: C It has been reported that i.v. administration of physostigmine salicylate may reverse some of the CNS and cardiovascular effects of tricyclic antidepressants. The dosage that has been recommended for adults is 1 to 2 mg in very slow i.v. injection. Since physostigmine has a short duration of action, administration may have to be repeated at 30 to 60 minute intervals particularly in life-threatening signs such as arrhythmias, convulsions, and deep coma recur or persist after the initial dose of physostigmine. Ref: Benzodiazepines remain the agents of choice in treating seizures. Phenytoin is no longer recommended because of its limited efficacy and possible pro-dysrhythmic effects. Phenobarbital may be used as a long-acting anticonvulsant. Benzodiazepines are also the treatment of choice for the extreme agitation or delirium that occasionally are observed because of the anticholinergic effects of TCAs. Physostigmine, an acetylcholinesterase inhibitor, is contraindicated in patients with TCA overdoses. Although physostigmine was previously advocated for relief of anticholinergic effects, it may cause bradycardia and asystole in TCA cardiotoxicity. Flumazenil, a benzodiazepine antagonist, is contraindicated, even in the presence of a benzodiazepine co-ingestion. Several case reports exist of patients with concomitant TCA overdoses who had seizures after the administration of flumazenil.

46 22. 19yo woman with 2 previous episodes of mania develops a major depressive episode.Which of the following is the most appropriate initial pharmacologic management? A. imipramine B. lithium C. divalproex D. venlafaxine E. venlafaxin&diva

47 The answer is: B the patient has had manic episodes and now has a depressive episode, it is a Bipolar Disorder. The initial treatment in Bipolar Disorder is always a mood stabilizer. Remember that in these cases if you give monotherapy with an antidepressant, the antidepressant could trigger a manic episode. So, the answer is B or C. According to the American Psychiatric Association: •   The first-line pharmacological treatment for bipolar depression is the initiation of either lithium or lamotrigine •   Antidepressant monotherapy is not recommended •   As an alternative, especially for more severely ill patients, some clinicians will initiate simultaneous treatment with lithium and an antidepressant. In patients with life-threatening inanition, suicidality, or psychosis, ECT also represents a reasonable alternative. ECT is also a potential treatment for severe depression during pregnancy. Ref: Am J Psychiatry 159:4, April 2002 Supplement Lithium :Classic bipolar I symptom pattern: euphoric mania and severe depressions Significant manic symptoms Need all the antidepressant you can get Suicide risk Very inexpensive valproate/Depakote : Need something strong and fast Male gender, and not afraid of weight gain risk Rapid cycling

48 23. A patient presents to your office who has a history of bipolar disorder.  He is currently depressed and on no medications.  He is otherwise well.  The most appropriate treatment is? A.   Electroconvulsive therapy B.   Valproic acid C.   Lithium carbonate D.   Fluoxetine E   Desipramine

49 The answer is: C Lithium Carbonate. In this case the patient is without treatment, so he should be treated as a patient not yet in treatment. Acute Depression Goals of Treatment: Achieve remission of the symptoms of major depression and return the patient to the usual levels of psychosocial functioning. Avoid precipitating a manic or hypomanic episode. For patients not yet in treatment for bipolar disorder, initiate either lithium or lamotrigine. lamotrigine/Lamictal Depression is the dominant symptom Rapid cycling Need all the antidepressant you can get afraid of weight gain As an alternative, especially for more severely ill patients, consider initiating treatment with both lithium and an antidepressant simultaneously (although supporting data are limited). Antidepressant monotherapy is not recommended because of the risk of precipitating a manic or hypomanic episode. Consider ECT for:             - Patients whit life-threatening inanition, suicidality, or psychosis or             - Severe depression during pregnancy For patients who suffer a breakthrough depressive episode while on maintenance treatment, optimize the medication dosage Ref: Quick Reference to the American Psychiatric Association Practice Guidelines for the Treatment of Psychiatric Disorders. Compendium 2004.

50 24. -what is true with respect to infantile autism?
A. it is more common than huntington's chorea B. about half of these children show symptoms of organic brain disorder C. incidence is highest among females of ashkenazy jews D. despite delay in language development intelligence is usually normal E. more prevalent in lower SES

51 The answer is: B Prevalence of HD in the United States is 5.15 cases per 100,000 persons. Autistic disorder and related conditions affect up to people per 10, ) Approximately 10% of children with a pervasive developmental disorder exhibit a known medical condition. 3) The male-to-female ratio is 3-4:1. Japanese studies often indicate the more common occurrence of autism in Japan than in other countries. The high rates of autism reported in many Japanese studies may reflect higher incidence and prevalence in Japan. Alternatively, since Japanese clinicians are highly skilled to diagnose autism, they may identify cases that are overlooked in other countries. 4) Most individuals with autism also manifest mental retardation, typically moderate mental retardation with intelligence quotients (IQs) between approximately 35 and approximately 50. Although often difficult to evaluate with intelligence tests, three quarters of children with autism function in the mentally retarded range. 5) Although autistic disorder was initially reported in children of high social class, subsequent research has established that autistic disorder afflicts all social classes equally.

52 25.You are counselling a 64-year-old woman who is concerned about her husband's increasing disinterest in sexual activity.  Which one of the following statements about sexual function is correct?      A.    Age impairs sexual capacity equally in men and women.       B.    Women remain clitorically responsive throughout their lifespan.      C.    Decreased sexual activity occurs as men age even though androgen levels remain stable.  D.    Most antihypertensive drugs will increase erectile function by promoting vasodilation.    E.    Thresholds for male sexual arousal are unchanged with age.

53 The answer is: B 1) An estimated 70 percent of men and 20 percent of women over age 60 years are sexually active; sexual activity is usually limited by the absence of an available partner. Longitudinal studies have found that the sex drive does not decrease as men and women age; in fact, some report an increased sex drive. Masters and Johnson reported sexual functioning among those in their 80s. Ref: Kaplan & Sadock. 2) The physical changes resultant from both ageing and declining estrogen levels, affect female sexuality. In addition to circulating estrogen levels falling at menopause, the nervous system and vascular system also decline with age. Loss of fat and glandular tissue, combined with lessened muscle tone and tissue elasticity, makes the breasts and other body regions more drooping and flabby. The cervix, uterus and ovaries shrink, the vagina becomes dry and the lining tissue thinner, and the clitoris, which retains its sensitivity, decreases in size. 3) Testosterone levels decline gradually in men, starting from approximately age 30, and this decline continues throughout life. Ref: 4) About 25% of cases of erectile dysfunction are caused by drugs, especially antihypertensives (most notably reserpine,  -blockers, guanethidine, and methyldopa), alcohol, cimetidine, antipsychotics, antidepressants, lithium, sedative-hypnotics, leuprolide, and hormones such as estrogen and progesterone. Ref: 5) With normal aging, persons require more time to become sexually aroused. Although some persons perceive this gradual slowing as a decline in function, others do not consider it an impairment because it merely results in men and women taking more time to achieve orgasm. Ref:

54 26.A previously healthy, 68-year-old woman develops auditory hallucinations.  She cannot provide many details but believes her mother is speaking to her.  She has difficulty cooperating during the interview and physical examination, which is unremarkable.  Which one of the following is the most likely diagnosis?     A.    complex partial seizures    B.    Alzheimer's disease     C.  adverse medication effect  D.    hyperthyroidism    E    peduncular hallucinosis

55 The answer is: B She is healthy, they don't mention any diagnosis or treatment in this patient. A Complex Partial Seizure is an epileptic attack that involves a greater degree of impairment or alteration of consciousness/awareness and memory than a simple partial seizure. It may involve automatisms (the unconscious repetition of simple actions, gestures or verbal utterances), and/or sensory changes. There can be some loss or memory (amnesia) surrounding the seizure event. Other patients may report a feeling of tunnel vision or dissociation, which represents a diminishment of awareness without full loss of consciousness. Still other patients can perform complicated actions, such as travel or shopping, while in the midst of a complex partial seizure. Hyperthyroidism major clinical features are weight loss (often accompanied by a ravenous appetite), fatigue, weakness, hyperactivity, irritability, apathy, depression, polyuria, and sweating. Additionally, patients may present with a variety of symptoms such as palpitations and arrhythmias (notably atrial fibrillation), dyspnea, loss of libido, nausea, vomiting, and diarrhea. In the elderly, these classical symptoms may not be present and they may present only with fatigue and weight loss leading to apathetic hyperthyroidism Neurological manifestations are tremor, chorea, myopathy, and periodic paralysis. Stroke of cardioembolic origin due to coexisting atrial fibrillation may be mentioned as one of the most serious complications of hyperthyroidism. Peduncular Hallucinations are formed visual images often associated with sleep disturbance and are caused by lesions in the midbrain, pons and diencephalon.

56 27.each of the following is true about post partum psychosis except;
A. most likely to occur after first child B.  may be characterized by homicidal thoughts C.  occurs in less than 1% of women D.  often characterized by severe depression E.  may require psychiatric hospitalization

57 The answer is: D In the revised fourth edition of the Diagnostic and Statistical Manual of Mental Disorders (DSM-IV-TR), the diagnosis of “brief psychotic disorder with postpartum onset” is given when psychosis occurs within 4 weeks of delivery and mood symptoms are not present. If the psychosis lasts longer than 1 month, then other diagnoses, such as schizophreniform disorder, should be considered. More typically, postpartum psychosis occurs together with mood symptoms. In such situations, the patient is given a diagnosis of a mood disorder with the specifier of postpartum onset. The incidence of postpartum psychosis is approximately 1 to 2 per 1,000 births, with psychotic symptoms appearing within the first week after childbirth in most individuals. The most important risk factor for developing a postpartum psychosis is previous history. The risk of puerperal relapse is as high as 70 percent. Other risk factors include primigravida, being unmarried, cesarean section, having a female child, and previous personal or family history of psychiatric illness. Fetal distress and offspring abnormalities (neurological abnormalities, cyanosis, neonatal polycythemia, thrombocytopenia) have also been identified as risk factors. There may be obsessional thoughts regarding violence to the child. However, infanticide is rare. Although rare, infanticide is more likely to occur in postpartum psychosis than in other postpartum psychiatric disorders. Ref: Sadock, Benjamin J., Sadock, Virginia A. Kaplan & Sadock's Comprehensive Textbook of Psychiatry, 8th Edition. Lippincott Williams & Wilkins 2005.

58 28. young patients comes to you with new development of tremors after taking antipsychotic medication .How will you treat? A. Lorazepam B. Lorazepine C. Amantadine D. Benzodiazepine

59 The answer is: C Anticholinergic and Antiparkinsonian agents are used to control the extra pyramidal side effects of antipsychotic agents, including acute dystonic reactions, neuroleptic induced parkinsonism, and akathisia. The anticholinergics are drugs of choice for acute dystonias and for drug-induced parkinsonism. Agents used: Anticholinergics:   Benztropine, Biperiden, Trihexyphenidyl. Antihistamine/Anticholinergic:   Diphenhydramine Dopamine/Agonist:   Amantadine Benzodiazepines aren't useful in the treatment of EPS.

60 29.The wife of a 45-year-old executive says that over the past 6 months her husband has been accusing her of having an extramarital affair. He has been phoning her work place, checking her mail and phone calls. She says that there is absolutely no truth to the allegation, but despite the efforts of herself and his family to reassure him, he continues to accuse her. Which one of the following is the likely cause?       A.   delusional disorder       B.   paranoid schizophrenia     C.  anxiety disorder D.   antisocial personality disorder     E    schizoid personality disorder

61 The answer is: A For the Differential Diagnosis, remember that the patients with Delusional Disorders have non-bizarre Delusions and no other symptoms. In Schizophrenia for a material part of at least one month (or less, if effectively treated) the patient has had 2 or more of:     - Delusions (only one symptom is required if a delusion is bizarre, such as being abducted in a space ship from the sun) - Hallucinations (only one symptom is required if hallucinations are of at least two voices talking to one another or of a voice that keeps up a running commentary on the patient's thoughts or actions). - Speech that shows incoherence, derailment or other disorganization - Severely disorganized or catatonic behavior Any negative symptom such as flat affect, reduced speech or lack of volition. In Personality Disorders the Personality Traits consist of enduring patterns fo perceiving, relating to, and thinking about the environment, other people and oneself. Personality patterns must be stable and date back to adolescence or early adulthood. Anxiety Disorders: several disorders fall into this category, including: Panic Disorder, Obsessive-Compulsive Disorder, Post-Traumatic Stress Disorder, Phobias (including Social Phobia also known as Social Anxiety Disorder), and Generalized Anxiety Disorder. These disorders can result in extensive anxiety and fear for an individual, are chronic and may become worse if not treated. Delusions are not a prominent part

62 30-26YO student has been well controlled on lithium for five years
30-26YO student has been well controlled on lithium for five years. His last hospitalization was for a severe psychotic depression. He is now complaining of a six-week history of dysphoria, low energy, lack of drive and sleepiness. Your most appropriate initial intervention should be: A. Stop lithium B. Do thyroid function test D. Start an antidepressant E. Start an antidepressant and neuroleptic F. Check his hemoglobin

63 The answer is: B Lithium Side Effects: Among the side effects may be nausea, loss of appetite, & mild diarrhea. They are common during the early weeks, but usually diminish with time. Patients may also experience dizziness & a fine tremor of the hands. Other potential side effects include weight gain, hypothyroidism, increased white blood cell count, skin rashes, & birth defects. The following are symptoms of Hypothyroidism: - Fatigue, loss of energy, lethargy, Weight gain, Decreased appetite, Cold intolerance, Dry skin, Hair loss, Sleepiness, Muscle pain, joint pain, weakness in the extremities, Depression, Emotional lability, mental impairment, Forgetfulness, impaired memory, inability to concentrate, Constipation, Menstrual disturbances, impaired fertility, Decreased perspiration, Paresthesia and nerve entrapment syndromes, Blurred vision, Decreased hearing, Fullness in the throat, hoarseness Lithium Toxicity: Toxic levels of lithium in the blood can cause vomiting, severe diarrhea, extreme thirst, weight loss, muscle twitching, abnormal muscle movement, slurred speech, blurred vision, dizziness, confusion, stupor, or pulse irregularities. Anemia: The symptoms and signs of anemia represent cardiovascular-pulmonary compensatory responses to the severity and duration of tissue hypoxia. Severe anemia (eg, Hb < 7 g/dL) can be associated with weakness, vertigo, headache, tinnitus, spots before the eyes, fatigability, drowsiness, irritability, and even bizarre behavior. Amenorrhea, loss of libido, GI complaints, and sometimes jaundice and splenomegaly can occur. Finally, heart failure or shock can result. The answers provided are: a) Stop Lithium – you should stop lithium if the patient had toxic levels of it, but the symptoms in this patient aren’t concordant with that diagnosis. b) Do Thyroid function test – among the side effects of lithium therapy is hypothyroidism. c) Start an antidepressant – this patient doesn’t meet the criteria for a Major Depressive Episode. d) Start an antidepressant and neuroleptic – the same as in c) e) Check his hemoglobin – there is no reason to think that this patient could have anemia.

64 31. A 33-year-old patient. A panic disorder with agoraphobia  Which one of the following drugs is LEAST likely to be useful in preventing panic attacks?       A.    clonazepam B.    paroxetine C.    alprazolam D.    buspirone E.    desipramine

65 The answer is: D Buspirone is not effective for Panic Disorder. The efficacy of buspirone HCl has been demonstrated in controlled clinical trials of outpatients whose diagnosis roughly corresponds to Generalized Anxiety Disorder (GAD). Many of the patients enrolled in these studies also had coexisting depressive symptoms and buspirone HCl relieved anxiety in the presence of these coexisting depressive symptoms

66 32. A 25-year-old woman with depression repeatedly talks about irrelevant topics when being interviewed by a psychiatrist.  Which one of the following symptoms is the patient demonstrating?    A.    suppression      B.    denial      C.    introjection    D.    resistance    E.    repression

67 Suppression Conscious act of controlling and inhibiting an unacceptable impulse, emotion, or idea; differentiated from repression in that repression is an unconscious process. Dealing with emotional stressors by deferred dealing with the stressor. For example, a worker finds that he is letting thoughts about a date that evening interfere with his duties; he decides not to think about plans for the evening until he leaves work. Denial Defense mechanism in which the existence of unpleasant realities is disavowed; refers to keeping out of conscious awareness any aspects of external reality that, if acknowledged, would produce anxiety. Dealing with emotional stressors by failing to recognize obvious implications or consequences of a thought, act, or situation. For example, a disabled person plans to return to former activities although it is evident it is virtually impossible. Used To avoid becoming aware of some painful aspect of reality Introjection:                The process of assimilation of the picture of an object (as the individual conceives the object to be).  For example, when a person becomes depressed due to the loss of a loved one, his feelings are directed to the mental image he possesses of the loved one. Dealing with emotional stressors by internalizing the values or characteristics of another person; usually someone who is significant to the individual in some way. For example, adopting the ideals of a charismatic leader in order to deal with feelings of one's own inadequacy. Features of External World ar taken and made part of the self.

68 The answer is: D Resistance:
            This defense mechanism produces a deep-seated opposition to the bringing of repressed (unconscious) data to awareness.  Through its operation, the individual seeks to avoid memories or insights which would arouse anxiety. Repression Freud's term for an unconscious defense mechanism in which unacceptable mental contents are banished or kept out of consciousness; important in normal psychological development and in neurotic and psychotic symptom formation. Freud recognized two kinds of repression: (1) repression proper, in which the repressed material was once in the conscious domain, and (2) primal repression, in which the repressed material was never in the conscious realm. Moving thoughts unacceptable to the Ego into the unconscious, where they cannot be easily accessed. Unconscious Forgetting

69 33. A 25-year-old man is hospitalized after weeks of worsening psychosis.  He is given thiothixene, an antipsychotic medication.  Five days later, he develops a fever of 39.8 °C, becomes delirious and lies stiffly in his bed.  His family reports that he had been physically well prior to his admission.  Which of the following diagnosis is of most immediate concern?     A.    tardive dyskinesia   B.    viral meningitis C.    neuroleptic malignant syndrome D.    ruptured cerebral aneurysm   E.    unsuspected opioid dependence

70 The answer is: C NMS is a heterogeneous syndrome that spans a broad severity continuum. The diagnosis is made on clinical grounds based on the presence of certain historical, physical, and laboratory findings. The diagnosis is confirmed, but not necessarily excluded, by the presence of the following criteria:           Recent treatment with neuroleptics within past 1-4 weeks Hyperthermia (above 38°C)         Muscular rigidity         At least 5 of the following:                 + Change in mental status                 + Tachycardia                 + Hypertension or hypotension                 + Diaphoresis or sialorrhea                 + Tremor                 + Incontinence                 + Increased creatinine phosphokinase (CPK) or urinary myoglobin                 + Leukocytosis                 + Metabolic acidosis                 + Exclusion of other drug-induced, systemic, or neuropsychiatric illness

71 year-old man was recently hospitalized in a psychiatric unit, where he was started on olanzapine. Upon visiting his new primary care physician for a routine physical examination prior to participating in a vocational rehabilitation program, his physician notices that his thoughts are quite illogical. For example, when she asks the young man what sort of employment he hopes to work toward, he answers, "if you subtract some yellow from the sky, it becomes greener." After a brief silence, he then states, "telephone bills should never exceed twenty dollars ... according to the Book of Numbers." Which mental status examination finding most accurately describes this patient's thought processes?   A. Clang associations  B. Concrete thinking  C. Loose associations  D. Tangential thoughts  E. Thought blocking

72 clang association Association or speech directed by the sound of a word rather than by its meaning; words have no logical connection; punning and rhyming may dominate the verbal behavior. Seen most frequently in schizophrenia or mania. "I'm not trying to make noise. I'm trying to make sense. If you can't make sense out of nonsense, well, have fun". concrete thinking Thinking characterized by actual things, events, and immediate experience, rather than by abstractions; seen in young children, in those who have lost or never developed the ability to generalize (as in certain cognitive mental disorders), and in schizophrenic persons. loosening of associations Characteristic schizophrenic thinking or speech disturbance involving a disorder in the logical progression of thoughts, manifested as a failure to communicate verbally adequately; unrelated and unconnected ideas shift from one subject to another. Loose of associations: the patient makes statements that lead to other statements in a very loose way, so that the associative leaps are unclear. There are clearly associations going somewhere in your patient’s mind, but you can’t make them out.

73 The answer is: C tangentiality Oblique, digressive, or even irrelevant manner of speech in which the central idea is not communicated. "What city are you from?", "Well, that's a hard question. I'm from Iowa. I really don't know where my relatives came from, so I don't know if I'm Irish or French". blocking Abrupt interruption in train of thinking before a thought or idea is finished; after a brief pause, the person indicates no recall of what was being said or was going to be said (also known as thought deprivation or increased thought latency). Common in schizophrenia and severe anxiety. Your patient begins to say something, then stops in midthought and forgets what he was going to say

74 36. YOUNG WOMAN SWALLOWED 18 TABS OF BDZ AFTER A FIGHT WITH HER BOYFRIEND SHE WAS DIZZY, AFTER 6hrS SHE IS OK AND WANT TO LEAVE: A.OBSERVE FOR 24 HRS. B.ADMIT HER AGAINST HER WILL C.LET HER GO D.SEND HER TO ADDICTION CLINIC

75 The answer is: C Toxicity, Benzodiazepine     * Admit patients with hemodynamic instability, coma, or respiratory depression to the ICU.     * Watch for signs of withdrawal in patients who have been taking BZDs chronically before overdose.     * Patients may be discharged if they remain asymptomatic 4-6 hours post ingestion. Those with mild toxicity may be observed in the emergency department until they recover.     * Transfer patients who may require more advanced care than is available in either the ED or inpatient setting.

76 37.What is the most common comorbid psychiatric diagnosis in patients with chronic pain?       
A.   anxiety disorder       B.   depressive disorder       C.   substance abuse disorder       D.   somatization disorder   E. conversion disorder

77 The answer is: B Mortality/Morbidity: Chronic Pain Syndrome (CPS) can affect patients in various ways. Major effects in the patient's life are depressed mood, fatigue, reduced activity and libido, excessive use of drugs and alcohol, dependent behavior, and disability out of proportion to impairment.     * Sternbach's 6 D's of CPS are as follows:           * Dramatization of complaints           * Drug misuse           * Dysfunction/disuse           * Dependency           * Depression           * Disability

78 Presence of Unexplained Physical Symptoms is Highly Correlated with Mood Disorders
Depressed patients often present with numerous physical complaints. As the number of physical complaints increases, so does the likelihood of a mood disorder.1 30% of patients with depression experience physical symptoms for more than 5 years before proper diagnosis.2 Key Points: This study indicates that as the number of physical complaints reported by a patient increases, so does the likelihood of a mood disorder. Patients with mood disorders often present with multiple physical symptoms only as we will see in the next study. References: Kroenke K. Spitzer RL. Williams JB. Linzer M. Hahn SR. deGruy FV 3rd. Brody D. Physical symptoms in primary care. Predictors of psychiatric disorders and functional impairment. Archives of Family Medicine. 3(9):774-9, 1994 Sep. OBJECTIVE: To examine how the type and number of physical symptoms reported by primary care patients are related to psychiatric disorders and functional impairment. DESIGN: Outpatient mental health survey. SETTING: Four primary care clinics. PATIENTS: One thousand adult clinic patients, of whom 631 were selected randomly or consecutively and 369 by convenience. MAIN OUTCOME MEASURES: Psychiatric disorders as determined by the Primary Care Evaluation of Mental Disorders procedure; the presence or absence of 15 common physical symptoms and whether symptoms were somatoform (ie, lacked an adequate physical explanation); and functional status as determined by the Medical Outcomes Study Short-form General Health Survey. RESULTS: Each of the 15 common symptoms was frequently somatoform (range, 16% to 33%). The presence of any physical symptom increased the likelihood of a diagnosis of a mood or anxiety disorder by at least twofold to three-fold, and somatoform symptoms had a particularly strong association with psychiatric disorders. The likelihood of a psychiatric disorder increased dramatically with increasing numbers of physical symptoms. The prevalence of a mood disorder in patients with 0 to 1, 2 to 3, 4 to 5, 6 to 8, and 9 or more symptoms was 2%, 12%, 23%, 44%, and 60%, respectively, and the prevalence of an anxiety disorder was 1%, 7%, 13%, 30%, and 48%, respectively. Finally, each physical symptom was associated with significant functional impairment; indeed, the number of physical symptoms was a powerful correlate of functional status. CONCLUSIONS: The number of physical symptoms is highly predictive for psychiatric disorders and functional impairment. Multiple or unexplained symptoms may signify a potentially treatable mood or anxiety disorder. Lesse S. Am J Psychother. 1983;37: This paper is a review of a 17-year study of 1,465 patients who had severe masked depression. It offers a description of the most frequently found symptoms and signs that are characteristic of patients with this disorder. A very distinctive syndrome is outlined. 1. Kroenke K, et al. Arch Fam Med. 1994;3: 2. Lesse S. Am J Psychother. 1983;37:

79 Chronic Painful Physical Conditions (CPPCs) Comorbid with MDD
Key Points: Patients with depression were 4 times as likely to have chronic painful physical conditions as those without depression. Patients with CPPCs were also more likely to suffer from depression. References: Ohayon MM. Schatzberg AF. Using chronic pain to predict depressive morbidity in the general population. Archives of General Psychiatry. 60(1):39-47, 2003 Jan. Ohayon: BACKGROUND: Pain syndrome is thought to play a role in depression. This study assesses the prevalence of chronic (>or= 6 months' duration) painful physical conditions (CPPCs) (joint/articular, limb, or back pain, headaches, or gastrointestinal diseases) and their relationship with major depressive disorder. METHODS: We conducted a cross-sectional telephone survey of a random sample of subjects from 15 to 100 years old representative of the general populations of the United Kingdom, Germany, Italy, Portugal, and Spain. Answers provided during telephone interviews using the Sleep-EVAL system were the main outcome measure. Interviews included questions about mental disorders and medical conditions. Data on painful physical conditions were obtained through questions about medical treatment, consultations, and/or hospitalizations for medical conditions and a list of 42 diseases. RESULTS: Of all subjects interviewed, 17.1% reported having at least 1 CPPC (95% confidence interval [CI], 16.5%-17.6%). At least 1 depressive symptom (sadness, depression, hopelessness, loss of interest, or lack of pleasure) was present in 16.5% of subjects (95% CI, 16.0%-17.1%); 27.6% of these subjects had at least 1 CPPC. Major depressive disorder was diagnosed in 4.0% of subjects; 43.4% of these subjects had at least 1 CPPC, which was 4 times more often than in subjects without major depressive disorder (odds ratio [OR], 4.0; 95% CI, ). In a logistic regression model, CPPC was strongly associated with major depressive disorder (OR: CPPC alone, 3.6; CPPC + nonpainful medical condition, 5.2); 24-hour presence of pain made an independent contribution to major depressive disorder diagnosis (OR, 1.6). CONCLUSIONS: The presence of CPPCs increases the duration of depressive mood. Patients seeking consultation for a CPPC should be systematically evaluated for depression. Ohayon MM, Schatzberg AF. Arch Gen Psychiatry. 2003;60:39-47.

80 38.A 36-year-old business consultant, married for 8 years, gainfully employed with a Masters in Business Administration (MBA) states that he has had, “No interest in sex for the past six months.” His medical, physical, and neurological examinations are unremarkable. Given the absence of any recent crises in his life, the most reasonable cause of his sexual disinterest is A. Catatonic schizophrenia B. Trichotillomania C. Sexual addiction D. Impotence E. Depression

81 The answer is: E The symptoms of this patient are classified as Sexual Desire Disorder. DSM-IV-TR divides Sexual Desire Disorders into two classes: hypoactive sexual desire disorder, characterized by a deficiency or lack of sexual fantasies and desire for sexual activity, and sexual aversion disorder, characterized by an aversion to and avoidance of genital contact with a sexual partner. The former condition is more common than the latter. Patients with desire problems often have good ego strengths and use inhibition of desire defensively to protect against unconscious fears about sex. Lack of desire can also result from chronic stress, anxiety, or depression. Abstinence from sex for a prolonged period sometimes suppresses the sexual impulse. Desire problems may also be an expression of hostility toward the partner or signal a deteriorating relationship. Impotence is classified as Male Erectile Disorder, and it’s essential feature is a persistent or recurrent inability to attain, or to maintain until completion of the sexual activity, an adequate erection. This patient doesn’t refer this problem. Depressive disorders are often associated with low sexual desire, and the onset of depression may precede, co-occur with, or be the consequence of the deficient sexual desire. Trichotillomani :A compulsion to pull out one's own hair

82 39.DOUGHTER DOES'NT WANT TO EAT OR GO TO NURSEY , ALL MAKE A DIAGNOSIS OF SEPARATION ANXIETY EXCEPT:
A. HER MOTHER WENT TO HOSP. BEFORE AND LEFT HER. B. HER FATHER HAS AGAROPHOBIA WHEN HE WAS YOUNG ND HE IS OVERANXIOUUS ABOUT HER C. HER BABY SITTER PUT HER IN DARK ROOM ALONE ONCE.

83 The answer is: C essential feature of Separation Anxiety Disorder is excessive anxiety concerning separation from the home or from those to whom the person is attached. Separation anxiety disorder may develop after some life stress. Separation from Primary Caregiver Present from 1-3 / Averge 7.5 at Onset – Prominent Physical symptoms, unrealistic Phobias an Fears, Sometimes evidence of nail biting and scratching (Self Mutilation) Treatment: Family Therapy, CBT, SSRI and BZD Children of parents with panic disorder have more than a threefold increase in the risk for SAD. In this case a) mother went to hospital and left her, constitute life stress for the child. b)The fact that her father has had agoraphobia and is overanxious is a risk factor for Separation Anxiety Disorder.  

84 40.19-year-old college student broke up with her boyfriend 72 hours ago. She now complains of depressed mood, anxiety, difficulty concentrating on studies, and preoccupation with "mistakes" she made in the relationship. Which one of the following is the most appropriate treatment at this time? A. desipramine B. diazepam C. phenelzine D. psychotherapy E. thioridazine

85 The answer is: D There are typically 5 stages of grief. These reactions do not occur in a specific order, and may (at times) show simultaneously. Not all of these emotions are necessarily experienced:     * Denial, disbelief, numbness     * Anger, blaming others     * Bargaining (e.g., "If I am cured of this cancer, I will never smoke again.")     * Depressed mood, sadness, and crying     * Acceptance, coming to terms Individuals who are grieving will frequently report crying spells, some trouble sleeping, and difficulty being productive at work. Starts within 3 months and less than 6 months Grief is a healthy response to loss that should not be prevented. Rather, it should be respected, and support should be provided to the grieving. Emotional support for the grieving process is usually provided by family and friends. Sometimes outside factors can influence the normal grieving process, and outside help from clergy, social workers, mental health specialists, or self-help groups may be indicated. The acute phase of grief can usually last up to 2 months, but some residual milder symptoms may extend a year or longer. Psychological counseling may benefit a person suffering from absent grief reaction, or from depression associated with grieving Treatment: Brief Psychotherapy, Crisis Intervention BZD, SSRI

86 1. 45-year-old business executive with advanced cirrhosis of the liver and a history of alcohol abuse claims that he does not have a problem with drinking and can quit any time he wants to. Eventually he quits drinking but continues to have the symptoms of advanced cirrhosis of the liver. Despite the obvious discomfort caused by his illness, he tells everyone how happy he is to have cirrhosis because it has led to the cessation of his drinking. The defense mechanism he is using is best identified as A) projection. B) denial. C) counter phobic behavior. D) reaction formation. E) isolation of affect

87 The answer is: D 1) Projection: Attributing to others one's own unacceptable thoughts, feelings, impulses, etc. So, the white person with repressed sexual urges may believe that all blacks are preoccupied with sex. The moralistic spouse, who is tempted to have an affair, begins to suspicion that his/her partner has been unfaithful. A slightly different form of paranoid projection is when a self-critical feeling or idea is attributed to others. Suppose a young woman from a religious family has strong feelings against any sexual urges she might have and, thus, almost never has them. She might start to believe, however, that others are critical of her whenever she wears a dress that shows her shape. 2) Denial: the individual deals with emotional conflict or internal or external stressors by refusing to acknowledge some painful aspect of external reality or subjective experience that would be apparent to others. Refusing to admit or face a threatening situation. Denial can be when a dying person refuses to admit what is going to happen or when a person with a heart condition denies that their overeating or smoking is of any consequence. In denial the individual cannot or will not face emotional conflict or turmoil related to internal or external stressors and based on this inability refuses to acknowledge aspects or the entire situation or event because it is too painful

88 3) Counter phobic behavior:  This defense mechanism involves deliberate approach of the object of one's fear or phobia. 4) Reaction formation: the individual deals with emotional conflict or internal or external stressors by substituting behavior, thoughts, or feelings that are diametrically opposed to his or her own unacceptable thoughts or feelings (this usually occurs in conjunction with their repression). It’s a denial and reversal of our feelings. Love turns into hate or hate into love. The feelings and actions resulting from a reaction formation are often excessive, for instance the loud, macho male may be concealing (from himself) sexual self-doubts or homosexual urges. Or, the person who is unconsciously attracted to the same sex may develop an intense hatred of gays. People, such as TV preachers, who become crusaders against "loose morals" may be struggling with their own sexual impulses.

89 The answer is: D 5) Isolation of Affect: the individual deals with emotional conflict or internal or external stressors by the separation of ideas from the feelings originally associated with them. The individual loses touch with the feelings associated with a given idea (e.g., a traumatic event) while remaining aware of the cognitive elements of it (e.g., descriptive details).

90 2. Most characteristic in HIV dementia is....
A) seizures and rigidity.. B) cognitive impairment C) ataxia and confusion D) pseudobulbar palsy

91 The answer is: B AIDS affect cognitive, behavioral, and motor function. •   Patients often present with the insidious onset of reduced work productivity, poor concentration, mental slowness, decreased libido, and forgetfulness. •   Apathy and withdrawal from hobbies or social activities are common and must be differentiated from depression. •   Rare features include sleep disturbances, psychosis (with mania), and seizures. •   Motor problems include imbalance, clumsiness, and weakness. •   Early signs and symptoms are subtle and may be overlooked. •   Later, these symptoms evolve into a global dementia with memory loss and language impairment. This can lead to a vegetative state.

92 3.In panic disorder , the most effective and long lasting treatment is :
A)   behavior treatment B)   psychoanalysis C)   imipramine D)   diazepam E)   hypnotherapy

93 Nowadays Cognitive Behavioral Therapy (CBT) is the most effective treatment in Panic Disorder. The Ontario Guidelines for the management of anxiety disorders in primary care says: No consistent differences have been shown in the efficacies of CBT, medications, and their combination over the short term. While many trials do not look at long term outcomes, there is a growing literature that supports better long term outcomes with fewer relapses with CBT compared to medications (Barlow, 2000; Marks, 1993). Little is known about the best way to sequence different treatments. If agoraphobia is present it is more important to include exposure as part of the CBT regimen. Antidepressants may enhance the effectiveness of exposure therapy. Including spouses in CBT improves response. Ref: Cognitive behavior therapy (CBT) is more effective and more cost-effective than medication. Tricyclic antidepressants (TCAs) and serotonin selective reuptake inhibitors are equal in efficacy and both are to be preferred to benzodiazepines. Treatment choice depends on the skill of the clinician and the patient’s circumstances. Drug treatment should be complemented by behavior therapy.

94 The answer is: A or C? Panic Disorder Treatment: Psychotherapy
Supportive Psychotherapy, relaxation techniques (Box-Breathing, visualization), CBT Pharmacotherapy BZD (short term, low dose, regular schedule, long half-life, NO PRN) SSRI/SNRI MAOIS Avoid Bupropion

95 4. 35 YO woman presents with the recent belief that since Kim Campbell was defeated she must take over as Prime Minister of Canada. She requires two hours of sleep per night, and feels wonderful and energetic. Which of the following symptoms is she LEAST likely to show? A)   Auditory hallucinations B)   Disorganized thinking C)   Disorientation for time and place D)   Easily angered and irritated E)   Poor insight

96 The answer is: C DDE of this Case; Schizoaffective Disorder
Manic episode with Psychotic features Schizophrenifom Disorder Bipolar I disorder with Psychotic features

97 5.The following therapies may be used in the management of chronic pain, EXCEPT :
A) operant conditioning. B) cognitive therapy. C) biofeedback. D) systematic desensitization. E) relaxation therapy.

98 •    Operant conditioning is a process of behavior modification in which a subject is encouraged to behave in a desired manner through positive or negative reinforcement, so that the subject comes to associate the pleasure or displeasure of the reinforcement with the behavior. •    Cognitive Therapy is a direct form of psychotherapy based on the interpretation of situations (cognitive structure of experiences) that determine how an individual feels and behaves. It is based on the premise that cognition, the process of acquiring knowledge and forming beliefs, is a primary determinant of mood and behavior. The therapy uses behavioral and verbal techniques to identify and correct negative thinking that is at the root of the aberrant behavior.

99 •    Biofeedback  uses electronic devices to measure physiological processes such as breathing rate; heart rate; skin temperature; skin conductance (which varies with perspiration); and muscle tension, which is measured by a process called surface electromyography (EMG). A biofeedback unit processes the electronic signals and “feeds back” the information to the user in the form of sounds, or graphs on a computer screen. There is no magic here; simply by practice a client can learn to alter these various physiological processes. The goal usually is to change them from a state of nervous arousal to a state of deep relaxation. •    Systematic Desensitization, used primarily to treat phobias related to one particular issue, is also a simple process whose effectiveness depends on practice. Essentially you create an anxiety hierarchy (a graded list of anxiety-provoking items) and then proceed to pair each item with the feeling of being deeply relaxed. Eventually this training process allows the patient to remain relaxed even when thinking about the anxiety-provoking situation. Finally, the patient learns to confront the real situation while remaining calm and relaxed.

100 The answer is: A •    Relaxation techniques: the patient learns to recognize the feeling of tension in his/her body and then he/she learns to release the tension. It’s a simple process, but it takes practice. There are three kinds of relaxation: Progressive Muscle Relaxation (in which the patient tense and relax various muscles, progressively, through his/her body), Guided Imagery (in which the patient visualizes relaxing images and situations), and Autogenics (in which the patient learns to create a feeling of warmth and heaviness throughout his/her body).

101 6 .30 years old being treated for his 1st episode of schizophrenia with haloperidol 10 mg daily and benzotropine 2mg daily is still feeling nervous and restless despite a reduction in his auditory hallucination what's your next step: A) increase the dose of haloperidol B) decrease the dose of haloperidol C) change to different class of antipsychotics D) increase the dose of benzotropine E) do supportive psychotherapy to help decrease his anxiety associated with stress of having such a serous illness

102 The answer is: B Akathisia is a drug induced reaction. The anticolinergic agents (as Benzotropine) are drug of choice for acute dystonias and for drug induced Parkinsonism. Anticholinergic are less effective for drug induced Akathisia, which often requires addition of a beta blocker. Acute within 10 days and tardive >90days benzotropine - For use as an adjunct in the therapy of all forms of parkinsonism. Useful also in the control of extrapyramidal disorders (except tardive dyskinesia) due to neuroleptic drugs (e.g. phenothiazines). Treatment: Lorazepam, Propanolol, diphenhydramine, reduce or change neuroleptice to lower potency

103 7-which of the following is the best treatment for agoraphobia with panic?
A)   Lorazepam 1mg tid B)   Fluxetine 20mg od C)   Fluphenazine 20mg od D)   Cognitive therapy E)   Lithium 330mg qid

104 The answer is: B Selective serotonin reuptake inhibitors and tricyclic antidepressants are equally effective in the treatment of panic disorder. The choice of medication is based on side effect profiles and patient preferences. Strong evidence supports the effectiveness of cognitive behavior therapy in treating panic disorder Benzodiazepines are effective in treating panic disorder symptoms, but they are less effective than antidepressants and cognitive behavior therapy Imipramine and clomipramine are considered first-line treatment options for panic disorder

105 8. Which one of the following types of hallucination is normal?       
A)    hypnagogic      B)    lilliputian    C)    tactile    D)    olfactory E)    auditory

106 The answer is: A Lilliputian: hallucination of reduced size of objects or persons. Hypnagogia (also spelled hypnogogia) and hypnopompia are experiences a person can go through when falling asleep in the case of hypnagogia, or waking up, in the case of hypnopompia. When in a hypnagogic or hypnopompic state a person can have lifelike auditory, visual, or tactile hallucinations (collectively known as hypnagogic hallucinations), perhaps even accompanied by full body paralysis. The individual is aware that these are hallucinations; the frightening part, in many cases, is the inability to react to them, even being unable to make a sound. In other cases one may enjoy truly vivid imaginations.

107 9-The most characteristic EEG finding in delirium is.............
A) Normal EEG B) Generalized low frequency, high amplitude theta waves C) Periodic complexes D) Focal, paroxysmal spikes E) Generalized 3-Hz spike and wave complexes

108 The answer is: B  In EEG characteristics of delirium include slowing or dropout of the posterior dominant rhythm, generalized theta or delta slow-wave activity, poor organization of the background rhythm, and loss of reactivity of the EEG to eye opening and closing. Toronto Notes: Generally Abnormal, slowing or fast activity

109 10.A 30-year-old male has been referred to a psychiatrist by his physician. The patient describes a constantly depressed mood with decreased interest in pleasure for the last 3 years. He is preoccupied by his state of mind, and he would like to have a prescription so that he can return to his normal mood. Although the patient describes about 2 or 3 episodes on “feeling high” and increased libido, he then returned to the constantly depressed mood after each of these episodes. He is an engineer and his mood did not cause interferences with his work. During the interview, the patient denies any suicidal thoughts or disturbances in appetite or sleep. His weight is in normal limits and there were no variations for the last 3 years. The patient also denies the use of drugs or medications. Physical examination and lab results are in normal limits. The most likely diagnosis is A) Major depressive disorder B) Bipolar I disorder C) Bipolar II disorder D) Dysthymic disorder E) Cyclothymic disorder

110 Bipolar I – At least one manic or mixed episode, could be accompanied by MDE but not necessary for DX Bipolar II – at least 1 MDE and at least 1 Hypomanic – No past manic or mixed episode

111 The answer is: E Numerous episodes of hypomanic and depressive Sx (not meeting criteria for MDE) for more than 2 years; never without Sx for more than 2 months The symptoms cause clinically significant distress or impairment in social, occupational, or other important areas of functioning. Treatment – anticonvulsants; psychotherapy

112 11.Which one of the following is the most common psychiatric complication of insulin-dependent diabetes mellitus? A. Hypochondriacally over-concern in later stages B. Major affective disorder C. Initial adjustment disorder with depressed mood D. Suicide attempts with the emergence of diabetic retinopathy E. Panic attacks

113 Adjustment disorders: Impact of adjustment disorders in those diagnosed with diabetes is most marked in case of children and adolescents with type 1 diabetes. Anxiety disorders: Anxiety is associated with poor glycemic control and treatment of anxiety is associated with improved glycemic control particularly in the subset of patients with more severe anxiety Sexual dysfunction: Data on sexual dysfunction in diabetes is complex. Apart from the ‘organic’ erectile dysfunction (attributable to complications of diabetes like neuropathy and vascular disease), other aspects of sexual functioning like desire, arousal, ejaculation, orgasm and satisfaction are also impaired in patients with diabetes.

114 The answer is: C Eating disorders: A recent large multi-site case-controlled study demonstrated that the prevalence rates of both full syndrome and sub-threshold eating disorders among adolescent and young adult women with diabetes are twice as high as in their non-diabetic peers. Factitious disorders: Factitious hypoglycemia as a manifestation of Munchausen’s syndrome, was assessed . in a study which detected a second generation of the sulfonylurea oral hypoglycemic agent in 17% of 129 patients who had unexplained severe hypoglycemia (Trenque et al., 2001). Similar situations relevant to diabetes treatment wherein patients present with hypoglycemic episodes (by voluntary overdosage of hypoglycemic drugs) or ketoacidosis (skipping insulin doses) have been reported

115 12-20-year-old college student presents to you with a history of intermittent chest pain for about 6 months. He states that the pain can occur at any time, but often occurs late in the day. The pain can be sharp, dull, or aching and may last for hours. He is physically active and is carrying a full academic load. The pain is unrelated to exercise and does not restrict his activities. His father and paternal uncle have both had myocardial infarctions within the past year. Physical examination is normal, although the patient appears restless and has a heart rate of 100/min. Which one of the following is the most likely diagnosis? A)    Reflux esophagitis B)    generalized anxiety disorder C)    transient myocardial ischemia D)    myocardial infarction E)    costochondritis

116 The answer is: B this young patient has multiple documented sources of anxiety dating back to last 6 months, e.g. full academic load, relatives having major diseases. He is also s/s of anxiety restlessness, increased heart rate etc. 3 or more of the following Restlessness or on edge Sleep disturbance Irritability Difficulty concentrating or mind going blanc Being easily fatigued Muscle Tension

117 year-old man with a long history of alcohol use comes to the emergency room of a general hospital.  He is confused and on examination is noted to have palsies of conjugate gaze, horizontal nystagmus, and ataxia.  Which one of the following is the most likely diagnosis?        A)    conversion reaction       B)    alcohol intoxication      C)    alcohol hallucinosis       D)    delirium tremens   E)  Wernicke's encephalopathy

118 DT is more common in patients with a long history of ethanol use and a prior history of significant withdrawal. Manifestations of ethanol withdrawal may start several hours to days after cessation or diminution of ethanol intake. Ethanol withdrawal seizures typically occur 6-48 hours after the last drink. DT usually begins hours after cessation or reduction of ethanol use. Symptoms may include the following: Tremors Irritability Insomnia Nausea/vomiting (frequently secondary to gastritis or pancreatitis) Hallucinations (auditory, visual, or olfactory) Confusion Delusions Severe agitation Physical: Tachycardia Hyperthermia Hypertension Tachypnea Diaphoresis Tremor Mydriasis Ataxia Altered mental status Hallucination Cardiovascular collapse

119 Consideration for Wernicke encephalopathy should be given to patients with any evidence of long-term alcohol abuse or malnutrition and any of the following: acute confusion, decreased conscious level, ataxia, ophthalmoplegia, memory disturbance, hypothermia with hypotension, and delirium tremens. Ocular abnormalities are the hallmarks of Wernicke encephalopathy. The oculomotor signs are nystagmus, bilateral lateral rectus palsies, and conjugate gaze palsies reflecting cranial nerve involvement of the oculomotor, abducens, and vestibular nuclei. Less frequently noted are pupillary abnormalities such as sluggishly reactive pupils, ptosis, scotomata, and anisocoria. Encephalopathy is characterized by a global confusional state, disinterest, inattentiveness, or agitation. Stupor and coma are rare. Gait ataxia is likely to be a combination of polyneuropathy, cerebellar damage, and vestibular paresis. Vestibular dysfunction without hearing loss is a common finding. In less severe cases, patients walk slowly with a broad-based gait. However, Hypothermia is common because of the involvement of the temperature-regulating center in the brainstem. Hypotension, caused by a defect in efferent sympathetic outflow and decreased peripheral resistance, may be present. Hypotension can also be the result of significant alcoholic liver disease. Coma is rarely the sole manifestation of Wernicke encephalopathy.

120 14.The initial management of a bipolar patient in the manic phase could include all of the following, EXCEPT        A)    neuroleptic       B)    low-stimulation environment.      C)    antidepressant       D)    lithium carbonate      E)    electroconvulsive therapy.

121 Mood Stabilizers: Antipsychotics: Haloperidol: 0.5-5 PO b/tid,
Lithium: Therapeutic Response within 7-14 days up to 3 weeks, mg/day - ) mmol/L ( mmol/L Acute) – Monitoring: Biweekly or Monthly, q2months – Thyroid and Creatinine 6 months, UA annual Carbamazepine (Tegretol): mg/day Rapid cycling, Severe sleep problems, Can't take Depakote lamotrigine/Lamictal: Depression is the dominant symptom, Dysphoric mood Valproic Acid (Depakene-Epival) valproate/Depakote: Need something strong and fast, Male gender, and not afraid of weight gain risk Rapid cycling, Significant manic symptoms Antipsychotics: Haloperidol: PO b/tid, Chlorpromazine (largactil): PO b/tid FDA:To control the manifestations of the manic type of manic-depressive illness, low potency Olanzapine/Zyprexa: Emergency-level symptoms, Need help really fast,Can use on "as-needed" basis

122 •   The first-line pharmacological treatment for bipolar depression is the initiation of either lithium or lamotrigine •   Antidepressant monotherapy is not recommended •   As an alternative, especially for more severely ill patients, some clinicians will initiate simultaneous treatment with lithium and an antidepressant. In patients with life-threatening inanition, suicidality, or psychosis, ECT also represents a reasonable alternative. ECT is also a potential treatment for severe depression during pregnancy. Lithium :Classic bipolar I symptom pattern: euphoric mania and severe depressions Significant manic symptoms Need all the antidepressant you can get Suicide risk Very inexpensive valproate/Depakote : Need something strong and fast Male gender, and not afraid of weight gain risk Rapid cycling

123 The answer is: C Acute Depression of Bipolar Goals of Treatment:
Achieve remission of the symptoms of major depression and return the patient to the usual levels of psychosocial functioning. Avoid precipitating a manic or hypomanic episode. For patients not yet in treatment for bipolar disorder, initiate either lithium or lamotrigine. lamotrigine/Lamictal Depression is the dominant symptom Rapid cycling Need all the antidepressant you can get afraid of weight gain As an alternative, especially for more severely ill patients, consider initiating treatment with both lithium and an antidepressant simultaneously (although supporting data are limited). Antidepressant monotherapy is not recommended because of the risk of precipitating a manic or hypomanic episode. Consider ECT for:             - Patients whit life-threatening inanition, suicidality, or psychosis or             - Severe depression during pregnancy For patients who suffer a breakthrough depressive episode while on maintenance treatment, optimize the medication dosage

124 15.The son of a 79-year-old woman wants a psychiatric evaluation for his mother because she has been more confused and lethargic during the past 6 months.  She takes several medications daily, although her son doesn't know what they are.  Mental status examination shows the woman to be dishevelled, mildly agitated, and delirious.  Which one of the following is the most useful next step in this woman's evaluation?        A)    identifying her medications       B)    eliciting a family history       C)    ordering serum electrolytes       D)    obtaining a CT scan of the head  E)    prescribing haloperidol for agitation

125 The answer is: A Dis-inhibition, impairment in ADL/IADL, personality changes, loss of social graces. ADL – IADL Activities of daily living Dressing, Eating, Ambulating, Toileting, Hygiene Instrumental Activities of daily living Shopping, Housekeeping, accounting, food preparation, transportation Toxin, MEDS: BZD, Beta Blockers, anticholinergics, heavy metals Indication for Head CT in dementia: Age<60 Rapid onset (unexplained decline in cognition or function over 1-2 months) Recent significant head trauma Unexplained neurological symptoms (new onset f several headaches/seizures)

126 year-old female has insomnia, irritability, increased psychomotor activity, and impulsivity. Which one of the following is the most likely diagnosis?        A)   Schizophrenia   B)   Antisocial personality disorder    C)   Major depressive disorder      D)   Hypomania     E)    Hyperthyroidism

127 The answer is: D Hypomanic episodes are characterized by the following: The patient has an elevated, expansive, or irritable mood of at least 4 days' duration. Three or more of the following symptoms are present: Grandiosity or inflated self-esteem Diminished need for sleep Pressured speech Racing thoughts or flight of ideas Clear evidence of distractibility Psychomotor agitation at home, at work, or sexually Engaging in activities with a high potential for painful consequences The mood disturbance is observable to others. The mood is not the result of substance abuse or a medical condition No psychotic features NOT SEVERE ENOUGH to cause social or occupational impairment, or to necessitate hospitalization

128 17.32-year-old man is brought into the emergency department by a friend because of a heroin overdose. After the administration of naloxone and a complete physical examination, which of the following laboratory studies is most important in the evaluation of this patient to provide long-term follow-up care?                 A. Albumin level              B. Echocardiogram              C. Electroencephalogram  D. HIV antibody test               E. Plasma liver enzyme levels

129 The answer is: D Major risk factors associated with the use of contaminated needles; Increased risk of Hep. B & C, Bacterial endocarditis, HIV Treatment: ABC IV Glucose Narcan : 0.4 mg up to 2 mg IV for diagnsis Intubation and mechanical ventilation, naloxene drip

130 18. A young boy was found behind a bar, delirious and agitated
18. A young boy was found behind a bar, delirious and agitated. Examination showed hypertension, tachycardia, muscle rigidity and loss of sensation to painful stimuli. The most likely diagnosis is A)   LSD B)   Alcohol C)   Heroin D)   Cocaine E)   PCP

131 The answer is: E Clinical Features of Phencyclidine Abuse: 1. Behavior changes include violence, belligerence, hyperactivity, catatonia, psychosis, anxiety, impairment of attention or memory, difficulty communicating. 2. Perceptual disturbances include paranoia, hallucinations, and confusion 3. Physical Examination:  Fever, diaphoresis, mydriasis Diagnostic Criteria for Intoxication: 1. Behavioral Changes 2. At least two of the following: nystagmus, hypertension or tachycardia, slurred speech, ataxia, decreased pain sensitivity, muscle rigididy, seizure or coma, hyperacusis (Abnormal hearing sensitivity)

132 19.Drug that is most associated with amnesia?
A) Phenytoin B) Lithium C) Lorazepam

133 The answer is: C Benzodiazepines (Valium, Diazepam) are a class of drugs which are primarily used to treat anxiety and insomnia. Benodiazepines are one of the most commonly prescribed classes of drugs sold. One of the problems associated with long-term use of these drugs, particularly in the elderly, is memory loss. Because aging is inherently associated with some degree of increasing memory impairment and because the elderly population is prone to insomnia, prescribing benzodiazepines is inadvisable. In fact, benzodiazepines are specifically administered to patients prior to surgery because they cause anterograde amnesia. Patients who have not been given a benzodiazepine drug preoperatively have complained about remembering their surgery, which leads to lawsuits.

134 20.Which one of the following is the most effective method of treatment for patients with pathological gambling disorders?      A)    activity group therapy   B)    self-help group therapy     C)    family therapy     D)    psychodrama    E)    individual therapy

135 Central to the problem and treatment of pathologic gambling is helping the patient overcome irrational thoughts. Pathologic gamblers believe they have the ability to control random or chance events by relying on superstitious behavior or methods. Treatment goals for patients who are pathologic gamblers or patients who are being treated for depression or alcoholism tend to be similar in that they focus on restoring a normal way of thinking and living to patients. A variety of approaches are used in the treatment of the pathologic gambler. Modeled after Alcoholics Anonymous, Gamblers Anonymous is the primary self-help group and uses a 12-step, abstinence-based treatment program. The efficacy of Gamblers Anonymous has not been demonstrated in controlled studies and, unlike alcoholism, some researchers have discovered that complete abstinence from gambling may not be necessary for successful treatment.

136 The answer is: E Behavioral, cognitive and cognitive-behavioral therapy appear to be the most successful treatment approaches. Pharmacotherapy appears to have a role in the treatment of coexisting depression, rather than as a primary treatment for pathologic gambling. According with this publication, the most effective method of treatment for patients with pathological gambling disorders would be individual psychotherapy. Ref:

137 21.Which of the following characteristics would be helpful in differentiating attention deficit  hyperactivity disorder from conduct disorder in children? A.   Resistance  to discipline B.   Temper tantrums C.   Distractibility E.   Aggressiveness F.   Truancy

138

139

140 The answer is: C

141 22.75ys man, hypertensive on methyldopa, diuretics and KCl supplement, benzotropine, and vitamins (including vitamin E), he was brought to the hospital. The nurse said that he was complaining of sleep disturbance for the last two days, and impaired memory, and he was brought now because of agitation. O/E he was disoriented, and confused, which drug is responsible for his confusional state? a)   methyldopa b)   vitamine E c)   benzotropine d)   diuretics e)   none of the above

142 The answer is: C Toxins, substance use, medication
Alcohol or alcohol withdrawal Anesthetics Anticholinergic Narcotics especialy morphine Sedative or sedative withawal Agents for Parkinson’s Anticonvulsants Steriods Insulin Glyburide Antibiotics, specially quinolones, NSAIDS SIDE EFFECTS - Benzotropines Nervous System: Toxic psychosis, including confusion, disorientation, memory impairment, visual hallucinations; exacerbation of pre-existing psychotic symptoms; nervousness; depression; listlessness: numbness of fingers

143 23.38-year-old woman tells her physician that for several months she has been experiencing palpitations, dizziness, shortness of breath, a feeling of impending doom and a fear of dying. Which of the following would be the most appropriate initial step in managing this patient?    A)    offer psychotherapy    B)    prescribe benzodiazepines    C)    perform a physical examination  D)    refer the patient to a psychiatrist E)    teach relaxation techniques

144 The answer is: C Panic disorder with or without agoraphobia must be differentiated from a number of medical conditions that produce similar symptomatology. Panic attacks are associated with a variety of endocrinological disorders, including both hypo- and hyperthyroid states, hyperparathyroidism, and pheochromocytomas. Episodic hypoglycemia associated with insulinomas can also produce panic-like states, as can primary neuropathological processes. These include seizure disorders, vestibular dysfunction, neoplasms, or the effects of both prescribed and illicit substances on the central nervous system (CNS). Finally, disorders of the cardiac and pulmonary systems, including arrhythmias, chronic obstructive pulmonary disease, and asthma, can produce autonomic symptoms and accompanying crescendo anxiety that can be difficult to distinguish from panic disorder. Clues of an underlying medical etiology to panic-like symptoms include the presence of atypical features during panic attacks, such as ataxia, alterations in consciousness, or bladder dyscontrol; onset of panic disorder relatively late in life; and physical signs or symptoms indicative of a medical disorder.

145 24. Chronic schizophrenic patient, on antipsychotic and antiparkinson therapy, develops tardive dyskinesia. What is the next step of management: a) Continue antipsychotic and increase antiparkinson drug b) Decrease and stop antipsychotic drug c) Continue anyipsychotic and decrease antparkinson drug d) Increase antipsychotic and stop antiparkinson drug

146 The answer is: B 1. All neuroleptics, with the exception of Clozapine, produce tardive dyskinesia. The risk of tardive dyskinesia with atypical antipsychotics is substantially decreased compared to typical agents 2. Antiparkinsonian drugs are of no benefit for tardive dyskinesias and may exacerbate symptoms. 3. When tardive dyskinesia symptoms are observed, the offending drug should be discontinued. Patients who require continued neuroleptic therapy should be switched to an atypical agent or Clozapine (if severe). 4. The risk of tardive dyskinesia increases with the duration of neuroleptic exposure, and there is an incidence of 3% per year with typical agents. 5. Most patients have relatively mild cases, but tardive dyskinesia can be debilitating in severe cases. Tardive dyskinesias not always improve with discontinuation or lowering of the does of neuroleptic

147 25.which of the following is correct about child abuse
a)   Children don’t lie about sexual or physical abuse b)   The parents were often victims of abuse themselves c)   It occurs is usually somebody the child does not know d)   It occurs mainly in the lower socio-economic class e)   You should not report your suspicions to the Children’s Aid Society unless there is ample evidence of abuse

148 The answer is: B The parents were often victims of abuse themselves

149 26- patient is convinced that an intravenous (IV) injection he received has made him immortal.  This is an example of which one of the following?     A)    an illusion   B)    a delusion  C)    a hallucination    D)    a perseveration E)    a projection

150 The answer is: B Illusion: misperception of a real external stimulus. Delusion is a belief about the world that most people would agree is impossible. A fixed false belief that is out of keeping with a person’s cultural or religious background and is firmly held despite incontrovertible proof to the contrary. Hallucination: sensory perception in the absence of external stimuli that is similar in quality to a true perception. Perseveration: your patient talks but dwells on a single idea or preoccupation over and over. This can be seen in both OCD and dementia, as well as in psychosis. Projection: is a defense mechanism, projection is falsely attributing to another his or her own unacceptable feelings, impulses or thoughts.

151 27. Common side effects of antipsychotics include all except
A) slurred speech B) anxiety and restlessness C) blurred vision D) nausea vomiting E) tremors

152 Patients ingesting antipsychotic medications, either short-term or long-term, often present to the ED with complaints of involuntary movement disorders. Dystonia, primarily manifesting as involuntary movement of the tongue, face, neck, or mouth Torticollis Oculogyric crisis Opisthotonus Hypotension and dysrhythmias may produce syncope, near syncope, orthostatic dizziness, and generalized weakness. Occasionally, patients present with a new onset seizure or are discovered in a postictal state. Dysrhythmia Phenothiazines are notorious for causing prolongation of the QT interval on the ECG and are associated with torsade de pointes. Other ECG findings include prolongation of the PR and QRS intervals and blocks. Similar to the anticholinergic effects of these medications, alpha blockade and postural hypotension may result in reflex tachycardia. Phenothiazines are associated with priapism caused by alpha blockade. Phenothiazines may cause photosensitivity, resulting in a blotchy red or purple discoloration of skin when it is exposed to sunlight. Anticholinergic syndrome: Toxic psychosis, agitation, confusion, mydriasis, urinary retention, ileus, hot flushed dry skin, and tachycardia may occur. Movement disorders Increased muscle tone, extrapyramidal symptoms, akathisia, restless legs, parkinsonism, or dystonia may occur. After chronic use of these medications (>24 mo), certain patients develop irreversible TD that consists of characteristic involuntary movements of the face, lips, and tongue. A disorder associated with intravenous use of prochlorperazine (Compazine) has been noted. Patients with this disorder become intensely anxious and restless and occasionally elope from the ED. These patients describe this acute dysphoric reaction as being very uncomfortable and creating the urge to crawl out of their skin. Whether this is an intense form of akathisia or a new movement disorder is unclear.

153 The answer is: B Neuroleptic malignant syndrome.
Miscellaneous abnormalities include metabolic acidosis, pulmonary edema, acute respiratory distress syndrome (ARDS), acute myocardial infarction, renal failure, pulmonary embolus, and disseminated intravascular coagulation (DIC). Virtually all neuroleptics produce some degree of extrapyramidal (EP) dysfunction because of inhibition of dopaminergic transmission in the basal ganglia. Several forms of extrapyramidal symptoms (EPS) are associated with neuroleptic toxicity. Acute dystonia Parkinsonism Akathisia: Motor restlessness and the urge to move are dose-related and occur in up to 20% of cases. Tardive dyskinesia All neuroleptics lower the seizure threshold to some degree, although certain ones (eg, chlorpromazine, clozapine, loxapine) have greater convulsant effects than others (eg, haloperidol, fluphenazine). The epileptogenic effect is dose-dependent, and the most common type of convulsion observed is a generalized tonic-clonic seizure. Adverse effects associated with chronic neuroleptic use include galactorrhea, priapism, cholestatic jaundice, skin photosensitivity, lens discoloration, and agranulocytosis.

154 28.the most dangerous withdrawl symptoms can be expected from:
A)diazepam  B)clonazepam  C)chlordiazepoxide  D)lorazepam  E)oxazepam

155 The answer is: E Half-lives: Diazepam             20 – 90 Clonazepam          Clordiazepoxide    Lorazepam           10 – 20 Oxazepam            4 – 14 Management: Tapering with long-acting BZD

156 29. Homeless 30-year-old man dressed in tattered clothes is brought into the Emergency Room by police after he was found haranguing passersby. Which one of the following is most suggestive of psychosis? A) hyper-religiosity and ascetic living habits B) rumination about the meaninglessness of material things C) a belief that his thoughts are controlled via secret television messages D) disorientation to time and place E) an unfounded suspicion that others are plotting against the government

157 The answer is: C A delusion of control or influence.The patient with delusions of control believes he is being controlled by some outside force

158 30.Affect is disturbed in all of the following except
A) Dysthymia B) hebephrenic schitzophrenia C) melancholic D) paranoid delusional psychosis

159 The answer is: D Dysthymia – A chronic disturbance of mood lasting at least two years in adults or one year in children, characterized by recurrent periods of mild depression and such symptoms as insomnia, tearfulness, and pessimism. Hebephrenic schitzophrenia - A form of schizophrenia in which affective changes are prominent, delusions and hallucinations fleeting and fragmentary, behaviour irresponsible and unpredictable, and mannerisms common. The mood is shallow and inappropriate, thought is disorganized, and speech is incoherent. There is a tendency to social isolation. Usually the prognosis is poor because of the rapid development of "negative" symptoms, particularly flattening of affect and loss of volition. Hebephrenia should normally be diagnosed only in adolescents or young adults. A syndrome characterized by shallow and inappropriate affect, giggling, and silly, regressive behavior and mannerisms; a subtype of schizophrenia now renamed disorganized schizophrenia. Melancholia - A mental disorder characterized by depression, apathy, and withdrawal. Paranoid delusional psychosis – Disturbances of affect, volition and speech, and catatonic symptoms, are either absent or relatively inconspicuous.


Download ppt "1.Which one of the following statements about the treatment of depression is correct:  A. Selective serotonin reuptake inhibitors (SSRIs) should be used."

Similar presentations


Ads by Google